Questions tagged [special-functions]

Many special functions appear as solutions of differential equations or integrals of elementary functions. Most special functions have relationships with representation theory of Lie groups.

Filter by
Sorted by
Tagged with
1 vote
0 answers
76 views

Conjectured closed form of $\int_0^1\frac{\ln^3(1+x)\,\ln^3x}x\mathrm{d}x$

I posted this question on Math Stack Exchange, but there were no helpful comments or answers https://math.stackexchange.com/q/4874446/1298448 How to integrate $${\displaystyle \int_0^1\frac{\ln^3(1+x)\...
Martin.s's user avatar
  • 194
4 votes
2 answers
584 views

Conjectured closed form of $\int\limits_0^1 \frac{\ln y \operatorname{Li}_2 (-y)}{1-y^2} \, dy$

Let's state with $\psi^{(1)}$ the trigamma. Calculate the order: $$\mathcal{S} = \sum_{n=1}^\infty (-1)^{n-1} (\psi^{(1)}(n))^2$$ (Cornel Ioan Valean) I uploaded this question here https://math....
Martin.s's user avatar
  • 194
1 vote
0 answers
63 views

Fitting a product into the quintuple or Jacobi triple product

The Rogers-Ramanujan functions fit nicely into the QPI or JTP. In fact we have that $$(q^{5};q^{5})_{\infty}(q,q^{4};q^{5})_{\infty}=\sum_{n=-\infty}^{\infty}(-1)^{n}q^{\frac{(5n^{2}-3n)}{2}}$$ and we ...
Jay's user avatar
  • 11
5 votes
1 answer
305 views

Conjectured closed form of $\int_0^1 \frac{\text{Li}_2\left(\frac{x}{4}\right)}{4-x}\,\log\left(\frac{1+\sqrt{1-x}}{1-\sqrt{1-x}}\right)\,dx$

After reading some meta posts, I've decided to post this question on MathOverflow since I didn't receive any comments or answers on MSE Certainly, I apologize for any oversight. Here's a more refined ...
Martin.s's user avatar
  • 194
5 votes
2 answers
3k views

Nicer expression for 2.1369288...?

In Drift Analysis and Evolutionary Algorithms Revisited by Johannes Lengler and Angelika Steger in Theorem 10, there is mention of a constant "$2.2$", and in the proof it becomes apparent ...
Moritz Firsching's user avatar
0 votes
0 answers
49 views

How to write Tricomi's confluent hypergeometric function in terms of Meijer-G function

I am calculating a closed form expectation and I encountered the Tricomi's confluent hypergeometric function (aka confluent hypergeometric function of the second kind) given by integral $U\left( a,b,z ...
K.K.McDonald's user avatar
2 votes
1 answer
166 views

Has the function $F_s(x)=\sum_{k=0}^{\infty}\frac{x^k}{\Gamma(k+1)^s}$ been studied before?

While studying an application of Grönwall's inequality I found that the function $$ F_s(x)=\sum_{k=0}^{\infty}\frac{x^k}{\Gamma(k+1)^s} $$ for $s\geq0$ in some cases provides a sharper bound. I had a ...
Felix Kastner's user avatar
1 vote
1 answer
161 views

Green's function for a linear PDE initial value problem

For $x\in\mathbb{R}^{n}$ and $t\in[0,\infty)$, consider the linear PDE initial value problem $$\dfrac{\partial u}{\partial t} = \left(a \Delta - \dfrac{b}{|x|}\right)u, \quad u(x,0) = u_0(x)\quad\text{...
Abhishek Halder's user avatar
6 votes
1 answer
157 views

About the high-order derivatives of Lambert function

In the mid seventies, in my former research group, we found that the $n^{\text{th}}$ derivative of $W_0(x)$ could write $$\frac {d^n\,W_0(x)}{dx^n}=(-1)^{n+1}\,\,\frac{\,P_n(w)}{ e^{nw}\,(1+w)^{2n-1}}\...
Claude Leibovici's user avatar
1 vote
1 answer
198 views

Antiderivative of Meijer G-function

In the python sympy CAS framework one strategy to compute integrals is to transform the integration kernel to a MeijerG-function, obtain the corresponding antiderivative as MeijerG-function and ...
maliesen's user avatar
  • 284
3 votes
1 answer
268 views

Where to find or how to prove that the ratio of two Bernoulli polynomials is increasing?

It is well known that the classical Bernoulli polynomials $B_j(t)$ are generated by \begin{equation*} \frac{s\operatorname{e}^{ts}}{\operatorname{e}^s-1}=\sum_{j=0}^{\infty}B_j(t)\frac{s^j}{j!}, \quad ...
qifeng618's user avatar
  • 828
14 votes
1 answer
676 views

Power series $x^n/n$ with one plus, then two minuses, then three plusses, and so on

Which function is represented by the powerseries $$1-\frac{x}{2}-\frac{x^2}{3}+\frac{x^3}{4}+\frac{x^4}{5}+\frac{x^5}{6}-\frac{x^6}{7}-\frac{x^7}{8}-\frac{x^8}{9}-\frac{x^9}{10} + \dots$$ (one plus, ...
user115748's user avatar
2 votes
1 answer
119 views

How to calculate this integral of squared Tricomi hypergeometric function

How to solve this integral $$ \int_{0}^{\infty}r^2 e^{-\omega r^2}U(-\nu,\frac{3}{2},\omega r^2)^2 \mathrm{d}r $$ where $\omega>0$ and $\nu \in \mathbb{R} \setminus \left \{ \frac{n-1}{2}\mid n \in ...
tsukatsuki_sorano's user avatar
21 votes
2 answers
2k views

Is the integral of $e^{-x^2}$ from $0$ to $1$ known to be irrational?

Is it known whether $$\int_0^1 e^{-x^2} \, dx$$ is irrational? It is well-known that $\int_0^\infty e^{-x^2} \, dx=\frac{\sqrt{\pi}}{2}$ which is irrational, but what about the prior integral? Also, I ...
Matthew Albano's user avatar
1 vote
1 answer
154 views

Derivation of indefinite integral involving hypergeometric function

I am doing a project on projectile motion and I ended up with this integral: $$\int \frac{m \left(g - \left(\frac{1}{e^t - g^{\frac{m}{c}}}\right)^{\frac{m}{c}}\right)}{c} \, dt$$ where $g, c,$ and $m$...
Leo McIntyre's user avatar
0 votes
1 answer
126 views

Asymptotic behavior of the polylogarithm function and generalisation

So, right now I am writing my master thesis and I need to find a reference for a formula I found in a paper: $$ \sum_{k=1}^{\infty}k^{-\alpha}(1-\varepsilon)^k\sim b+c\Gamma(1-\alpha)\varepsilon^{\...
yannik0103's user avatar
4 votes
1 answer
82 views

Eigenvalues of the modified Mathieu equation with normalizable solution

The Mathieu equation (https://en.wikipedia.org/wiki/Mathieu_function) is $y''+(a-2q\cos(2z))y=0$. The modified Mathieu equation is obtained by replacing $z$ with $\pm iz$: $$y''-(a-2q\cosh(2z))y=0.$$ ...
renphysics's user avatar
-1 votes
1 answer
124 views

Is there a single function describing this piecewise function?

I am examining the piecewise function given by the following. $f(x) = \pi - \arctan(\frac{2x}{1-x^2})$ when $0 \leq x < 1$, $f(x) = \frac{\pi}{2}$ when $x=1$, and $f(x) = \arctan(\frac{2x}{x^2-1})$ ...
Benjamin L. Warren's user avatar
6 votes
1 answer
365 views

When are the chirp signals orthogonal?

Assume that we have two bounded-time chirp signals, \begin{align} x(t)&=\exp\Big(j\pi(\alpha t^2+\beta t+\gamma)\Big),\quad 0\leq t\leq T,\\ y(t)&=\exp\Big(j\pi(\alpha' t^2+\beta' t+\gamma')\...
Math_Y's user avatar
  • 311
4 votes
1 answer
273 views

Double q-analog of Pochhammer

Has the function $$(z;q_1,q_2)_\infty := \prod_{n_1,n_2=0}^\infty (1-z \, q_1^{n_1} q_2^{n_2}), \quad |q_1|,|q_2|<1$$ been studied in the math literature? For example, does it obey any difference ...
jj_p's user avatar
  • 533
1 vote
2 answers
343 views

A closed formula for a sum involving hypergeometric function

Let ${ }_1 F_1(a ; c ; z)$ be Kummer's function defined by the function, and all its analytic continuations, represented by the infinite series $\sum_{n=0}^{\infty} \frac{(a)_n}{(c)_n} \frac{z^n}{n !...
zoran  Vicovic's user avatar
3 votes
1 answer
296 views

How to find partial derivatives of the Beta Function?

I was reading the book (Almost) Impossible Integrals, Sums and Series. The author used a method involving taking partial derivatives of the Beta Function to solve some integrals. $$B(x,y)=\int_0^1u^{x-...
Souparna's user avatar
  • 141
1 vote
1 answer
172 views

T functions arising from derivatives of incomplete Gamma function

Here the derivatives of the incomplete gamma functions are described via: $$ T(m,s,x) = G_{m-1,\,m}^{\,m,\,0} \!\left( \left. \begin{matrix} 0, 0, \dots, 0 \\ s-1, -1, \dots, -1 \end{matrix} \; \right|...
user18722294's user avatar
2 votes
1 answer
176 views

Sum of Bessel function with integer parameters and fixed argument

Question. Let $J_{\nu}$ be a standart Bessel function of the first order. What is the asymptotic of the sum $\sum_{n\ge 0}|J_n(t)|$ as $t\to\infty$? An upper bounds stronger than $O(t)$ are also ...
Pavel Gubkin's user avatar
1 vote
0 answers
74 views

How to calculate the Integral with confluent hypergeometric function

How to prove this.Thank you in advance Let $\delta,\beta>0$ How to prove this \begin{align} & \int^1_0 \frac{w^{1-\beta}}{(1-w)^{1+\delta}} (-t.s w)^{\frac{-\delta}{2}} e^{-\frac{w}{1-w}(s+t)}...
zoran  Vicovic's user avatar
9 votes
1 answer
620 views

What is the value of $j(2\sqrt{-163})$?

My question is how to calculate the value of $j(2\sqrt{-163})$ and its minimal polynomial, where the $j$ is elliptic modular function (see https://mathworld.wolfram.com/j-Function.html). The class ...
GuoJi's user avatar
  • 245
3 votes
2 answers
383 views

Functional equations based on composition

I have asked this question here (*), but there are no answer. Let $n \in \mathbb N^*$, $\{a_0,\ldots,a_n\} \subset \left] 0,+\infty\right]$. We suppose $Eq : \sum\limits_{k=0}^n a_k f^k(x)=0$ have no ...
Dattier's user avatar
  • 3,737
0 votes
0 answers
151 views

How to prove negativity of a $3\times3$ determinant whose elements involve trigamma, tetragamma, and pentagamma functions?

The classical Euler gamma function can be defined by the integral \begin{equation*} \Gamma(z)=\int_0^{\infty}t^{z-1}\operatorname{e}^{-t}\operatorname{d}t, \quad \Re(z)>0. \end{equation*} Its ...
qifeng618's user avatar
  • 828
0 votes
1 answer
126 views

Is $\Gamma(z,1)\not=0$ for all $z$ with $\Re(z)<0$?

I found this paper online which appears to present zeros of the incomplete gamma function within the right half plane. It makes me think that there are no zeros in the left half plane. Not sure how to ...
Bobby Ocean's user avatar
5 votes
0 answers
101 views

Ratio of theta functions as roots of polynomials

I already asked the same question here, but received no answer. I did some little progress and so I'm asking again. I was playing with the theta functions with argument $ z = 0 $ $ \vartheta_2(q) =\...
user967210's user avatar
7 votes
0 answers
214 views

Analytic continuation of Dixon's identity

Many well-known combinatorial identities has an analytic version. For example, the following identities $$ 2^n = \sum_{k=0}^n \binom{n}{k} $$ $$ \binom{2n}{n} = \sum_{k=1}^n \binom{n}{k}^2 $$ can be ...
Pluviophile's user avatar
  • 1,404
0 votes
0 answers
56 views

Inverse Laplace transform of the hypergeometric function 2F1

In the book Integrals and Series: Inverse Laplace Transforms by A.P. Prudnikov, the inverse Laplace transform of the hypergeometric function 2F1 defined as $$ _{2}F_{1}(p+a,b;c;-\omega), \qquad [b,p+a\...
Dante's user avatar
  • 251
0 votes
0 answers
36 views

Bessel functions of matrix argument in the scalar case

Herz (1955) provides the following equality: $$ A_\delta(-\lambda) - A_\delta(-\lambda)\lambda^{-\delta} = -\sin(\pi\delta)B_\delta(\lambda)/\pi $$ where $A_\delta$ and $B_\delta$ are the Bessel ...
Stéphane Laurent's user avatar
5 votes
0 answers
236 views

Questions on Gauss's geometric interpretation of spherical functions

(This question was initially posted on HSM stackexchange, but eventually I came to conclusion that it is too mathematical to be answered there.) In the physics chapter of his biography of Gauss, W.K. ...
user2554's user avatar
  • 1,869
3 votes
0 answers
154 views

Hilbert's 13th Problem and series solutions for the reduced sextic, septic, and octic?

I. Reduced equations One can eliminate 3 terms from the general quintic, sextic, septic, and octic using a Bring-Jerrard transformation to get the reduced forms in radicals, $$x^5+(x+p) = 0$$ $$x^6+(x+...
Tito Piezas III's user avatar
14 votes
6 answers
2k views

Closed form of an infinite series

Does the following infinite series have a closed form? $$ \sum_{n=1}^{\infty} {(-1)^n \frac{\Gamma(\frac{1}{3}+\frac{n}{3})}{\Gamma(1+\frac{n}{3})} \sin(\frac{2\pi n}{3})} $$
Dante's user avatar
  • 251
2 votes
0 answers
212 views

Zeta function associated with a function $f$

Let the function $f(t) = \cos(at)$, where ($0 < a < 1$). Let us define $$\zeta(z, f) = \frac{1}{\Gamma(z)} \int_0^{+\infty} \frac{t^{z-1}\cos(at)}{e^t-1}\, dt. $$ Is there a general formula that ...
L.L's user avatar
  • 399
7 votes
2 answers
515 views

Weak convergence related to Hermite polynomial?

I am reading Griffiths's quantum mechanics book; in the section about harmonic oscillators, he wrote out the amplitude of wave function, and compared with the classical harmonic oscillators. He ...
Yuval's user avatar
  • 637
2 votes
3 answers
433 views

Is the hypergeometric function ${}_1F_2(1;a,a+\frac12;-x^2)$ an elementary function? How about its positivity, monotonicity, and convexity in $x$?

Is the generalized hypergeometric function ${}_1F_2\bigl(1;a,a+\frac12;-x^2\bigr)$ for $a>-1$ and $x>0$ an elementary function? How about the positivity, monotonicity, and convexity of the ...
qifeng618's user avatar
  • 828
2 votes
1 answer
199 views

An integral transform computation

In Erdelyi, Tables of Integral Transforms, p. 344 Section 7.2. they note that \begin{align} \frac{1}{2 \pi i} \int_{c-i\infty}^{c+i\infty} s^{\nu} e^{\alpha s^2} x^{-s} \, ds = 2^{-\nu/2} \pi^{-...
user506603's user avatar
10 votes
7 answers
850 views

$\int_L^\infty \exp(- t - y/t) \, dt = \text{?}$

Let $y>0$, $L>0$. Has the (special?) function given by $$f(y,L) = \int_{L}^\infty e^{- t - y/t} \, dt$$ been studied? Are there precise, simple bounds? Let me try to attempt to reinvent the ...
H A Helfgott's user avatar
  • 19.3k
2 votes
0 answers
148 views

Finding similar Zudilin-Cohen recurrence relations and cfracs for $\frac{\zeta(4)}{13}$?

I. Two recurrence relations The first one was also discussed in this MO post. We have the similar, \begin{align} (n+1)^5 u_{n+1} &= (2n + 1)(9n^2 + 9n + 3)(15n^2 + 15n + 4)u_n +3n^3(9n^2-1)u_{n-1}\...
Tito Piezas III's user avatar
6 votes
1 answer
250 views

On the continued fractions using Cooper's sequences $s_7,\, s_{10},\, s_{18}$ and the Zudilin-Cohen sequence

In a previous MO post, H. Cohen suggested Gorodetsky's 2021 paper which discussed $6+6+3=15$ "sporadic sequences". The first 6 are Zagier's sporadic sequences, the second 6 are by Almkvist-...
Tito Piezas III's user avatar
-2 votes
1 answer
159 views

Two-variable continuous function which results in an integer if and only if arguments are integer

I am looking for functions $f(x,y)$, real arguments, continuous, with the following properties: $f(m,n) = r$, where $r$ is integer $> 0$ if and only if $m,n$ are integers $> 0$. $f(m,n) \le f(...
Jada's user avatar
  • 3
2 votes
2 answers
352 views

Finding a new level-$7$ pi formula using the relation $j_{7A}(\tau) = \left(\sqrt{j_{7B}(\tau)}+\frac{7}{\sqrt{j_{7B}(\tau)}}\right)^2$

(Note: This third method continues from this post.) There are level-$7$ pi formulas based on the McKay-Thompson series $T_{7A}$ and Cooper's $s_7$ sequence in this paper. This third method, among ...
Tito Piezas III's user avatar
10 votes
1 answer
433 views

On Ramanujan's pi formula $\frac 1\pi=\sum_{k=0}^\infty\frac {(4k)!}{k!^4}\frac {Ak+B}{396^{4k}}$ and the solvable quintic $z^5-5z-396 = 0$?

I. Four quintics? The general quintic can be transformed in radicals to at least three one-parameter forms. For simplicity, assume this free parameter to be some generic "alpha". Hence, $$x^...
Tito Piezas III's user avatar
4 votes
2 answers
494 views

Finding a sextic analogue to the solvable octic $\frac{(x + 1)^6(x^2 + x + 7)}x = -k^3$ where $e^{(\pi/3)\sqrt{d}}\approx k^3+41.999999999999\dots$

I. Degree 8 Assume the $j_i$ to be free parameters. The following octics in $x$ belong to $8T43,$ have group $\text{PGL}(2,7)$, and order $2\times168 = 336.$ \begin{align} {j_1}\; &=\frac{(x^2 + ...
Tito Piezas III's user avatar
2 votes
0 answers
66 views

Transforming a Fuchsian equation with four finite singularities to the Heun equation

I keep seeing it claimed that the general second-order Fuchsian equation with four singularities can be transformed to the Heun equation, but I have never seen anyone explicitly write out the steps, ...
J. M. isn't a mathematician's user avatar
2 votes
0 answers
153 views

What is known about "anti polynomials"?

I recently encountered a problem whose solution required solving $f(x):=\sum\alpha_i r_i^x\, =\, c;\ \alpha_i,r_i,x\in\mathbb{R},i\in I\subset\mathbb{N}, $ for $x$. While the Newton method solves the ...
Manfred Weis's user avatar
  • 12.6k
1 vote
0 answers
162 views

Ask for a proof of an inequality involving the Bernoulli numbers

Let $B_k$ be the Bernoulli numbers and let \begin{equation} T_k=\frac{2^{2k}}{(2k)!}|B_{2k}|, \quad k\ge1. \end{equation} Prove the inequality \begin{equation*} \frac{\frac{1}{k+2}\sum_{j=0}^{k+1}\...
qifeng618's user avatar
  • 828

1
2 3 4 5
17